Advertisement
KiraKuznetsova

Моя задача

May 8th, 2019
102
0
Never
Not a member of Pastebin yet? Sign Up, it unlocks many cool features!
text 2.40 KB | None | 0 0
  1. \documentclass{article}
  2. \usepackage{cmap}
  3. \usepackage[T2A]{fontenc}
  4. \usepackage[utf8]{inputenc}
  5. \usepackage[14pt]{extsizes}
  6. \usepackage[english,russian]{babel}
  7. \usepackage{euscript}
  8. \usepackage{amsmath,amsfonts,amssymb,amsthm,mathtools}
  9. \usepackage{esvect}
  10.  
  11. \title{НИС}
  12. \author{kirakuznetsova}
  13. \date{April 2019}
  14.  
  15. \begin{document}
  16.  
  17.  
  18. \section{Собственная задача}
  19. \textbf{Условие.} Докажите, что для любых положительных чисел $a,b,c$, таких что $a+b+c=9$, выполняется неравенство: $$\frac{a^2+b^2+c^2}{\sqrt{a}+\sqrt{b}+\sqrt{c}}\geq3\sqrt{3}$$
  20. \textbf{Решение.} Рассмотрим функцию $f(x)=x^2$. Вторая производная $f^{\prime\prime}(x)=2$ всегда положительна, а значит эта функция выпукла вниз. Запишем неравенство Йенсена для нее:
  21. $$f\left(\sum_{i=1}^n\alpha_ix_i\right)\leq\sum_{i=1}^n\alpha_if(x_i)$$
  22. По условию $n=3$, $x_1=a$, $x_2=b$, $x_3=c$, $a+b+c=9$, также примем $\alpha_i=\frac{1}{n}=\frac{1}{3}$. Учитывая это, наше неравество можно представить в виде:
  23. $$\left(\frac{a+b+c}{3}\right)^2\leq\frac{a^2+b^2+c^2}{3}$$
  24. $$a^2+b^2+c^2\geq\frac{1}{3}(a+b+c)^2=27$$
  25. Рассмотрим функцию $f(x)=\sqrt{x}$. Вторая производная $f^{\prime\prime}(x)=-\frac{1}{4\sqrt{x^3}}$ отрицательна на интервале $(0;+\infty)$. Следовательно, эта функция выпукла вверх. Запишем неравенство Йенсена для нее:
  26. $$f\left(\sum_{i=1}^n\alpha_ix_i\right)\geq\sum_{i=1}^n\alpha_if(x_i)$$
  27. По условию $n=3$, $x_1=a$, $x_2=b$, $x_3=c$, $a+b+c=9$, также примем $\alpha_i=\frac{1}{n}=\frac{1}{3}$. Учитывая это, наше неравество принимает такой вид:
  28. $$\sqrt{\frac{a+b+c}{3}}\geq\frac{\sqrt{a}+\sqrt{b}+\sqrt{c}}{3}$$
  29. $$\sqrt{a}+\sqrt{b}+\sqrt{c}\leq\sqrt{3}\sqrt{a+b+c}=3\sqrt{3}$$
  30. Из двух, получившихся у нас, неравенств следует, что:
  31. $$3\sqrt{3}(a^2+b^2+c^2)\geq27(\sqrt{a}+\sqrt{b}+\sqrt{c})$$
  32. $$\frac{a^2+b^2+c^2}{\sqrt{a}+\sqrt{b}+\sqrt{c}}\geq3\sqrt{3}$$
  33. Утверждение доказано.$\blacksquare$
  34. \end{document}
Advertisement
Add Comment
Please, Sign In to add comment
Advertisement